Difference between revisions of "2017 AMC 12A Problems/Problem 20"
(→Solution) |
m (→Solution) |
||
Line 7: | Line 7: | ||
==Solution== | ==Solution== | ||
− | By the properties of logarithms, we can rearrange the equation to read <math>2017 \log_b a=(\log_b a)^{2017}</math>. Then, subtracting <math>2017\log_b a</math> from each side yields <math>(\log_b a)^{2017}-2017\log_b a=0</math>. We then proceed to factor out the term <math>\log_b a</math> which results in <math>(\log_b a)( | + | By the properties of logarithms, we can rearrange the equation to read <math>2017 \log_b a=(\log_b a)^{2017}</math>. Then, subtracting <math>2017\log_b a</math> from each side yields <math>(\log_b a)^{2017}-2017\log_b a=0</math>. We then proceed to factor out the term <math>\log_b a</math> which results in <math>(\log_b a)((\log_b a)^{2016} -2017)=0</math>. Then, we set both factors equal to zero and solve. |
<math>\log_b a=0</math> has exactly <math>199</math> solutions with the restricted domain of <math>[2,200]</math> since this equation will always have a solution in the form of <math>(1, b)</math>, and there are <math>199</math> possible values of <math>b</math> since <math>200-2+1 = 199</math>. | <math>\log_b a=0</math> has exactly <math>199</math> solutions with the restricted domain of <math>[2,200]</math> since this equation will always have a solution in the form of <math>(1, b)</math>, and there are <math>199</math> possible values of <math>b</math> since <math>200-2+1 = 199</math>. | ||
− | We proceed to solve the other factor, <math>(\log_b a)2016-2017</math>. We add <math>2017</math> to both sides, and take the <math>2016th</math> root, this gives us <math>\log_b a=\sqrt[2016]{2017}</math> <math>\sqrt[2016]{2017}</math> is a real number, and therefore <math>a=b^{\sqrt[2016]{2017}}</math> Again, there are <math>199 \cdot 2 = 398</math> solutions, as <math>b^{\sqrt[2016]{2017}}</math> must be a real number (It's a real number raised to a real number), and we also have to multiply by two because when we take the <math>2016th</math> root, we must also consider the negative root which is valid because the taking the reciprocal of <math>a</math> negates <math>log_ba</math>. | + | We proceed to solve the other factor, <math>(\log_b a)^{2016}-2017</math>. We add <math>2017</math> to both sides, and take the <math>2016th</math> root, this gives us <math>\log_b a=\sqrt[2016]{2017}</math> <math>\sqrt[2016]{2017}</math> is a real number, and therefore <math>a=b^{\sqrt[2016]{2017}}</math> Again, there are <math>199 \cdot 2 = 398</math> solutions, as <math>b^{\sqrt[2016]{2017}}</math> must be a real number (It's a real number raised to a real number), and we also have to multiply by two because when we take the <math>2016th</math> root, we must also consider the negative root which is valid because the taking the reciprocal of <math>a</math> negates <math>log_ba</math>. |
Therefore, the answer is <math>199 + 398 = \boxed{\textbf{(E) } 597}</math>. | Therefore, the answer is <math>199 + 398 = \boxed{\textbf{(E) } 597}</math>. |
Revision as of 19:56, 8 February 2017
Problem
How many ordered pairs such that is a positive real number and is an integer between and , inclusive, satisfy the equation
Solution
By the properties of logarithms, we can rearrange the equation to read . Then, subtracting from each side yields . We then proceed to factor out the term which results in . Then, we set both factors equal to zero and solve.
has exactly solutions with the restricted domain of since this equation will always have a solution in the form of , and there are possible values of since .
We proceed to solve the other factor, . We add to both sides, and take the root, this gives us is a real number, and therefore Again, there are solutions, as must be a real number (It's a real number raised to a real number), and we also have to multiply by two because when we take the root, we must also consider the negative root which is valid because the taking the reciprocal of negates .
Therefore, the answer is .
Note: the former solution was incorrect because when we take the root, we must also consider the negative root which is valid because the taking the reciprocal of negates . Therefore the answer is or .
See Also
2017 AMC 12A (Problems • Answer Key • Resources) | |
Preceded by Problem 19 |
Followed by Problem 21 |
1 • 2 • 3 • 4 • 5 • 6 • 7 • 8 • 9 • 10 • 11 • 12 • 13 • 14 • 15 • 16 • 17 • 18 • 19 • 20 • 21 • 22 • 23 • 24 • 25 | |
All AMC 12 Problems and Solutions |
The problems on this page are copyrighted by the Mathematical Association of America's American Mathematics Competitions.